Limite, somme de Riemman

factoriel

Le problème exposé dans ce sujet a été résolu.

Salut à tous ! :)

J'ai eu un partiel où il y avait quelques limites à calculer dont 2 que je n'ai pas réussi à calculé et même là avec la correction j'ai du mal à bien saisir le truc…

Je vais essayer de tout bien écrire en Tex.

Voici la première :

$$ \lim_{n\to +\infty} \left(\frac{(4n)!}{n^{n}\times (3n)!}\right)^{\frac{1}{n}} $$

Je vous montre mon calcul : J'ai écrit que cette limite est égale à cela :

$$=\exp(\dfrac{1}{n} \times (\ln(4n!)-\ln(3n!)-n \times \ln(n) )$$

Or :

$$\ln(4n!)= \sum_{k=1}^{4n} \ln(k)$$
$$\ln(3n!)= \sum_{k=1}^{3n} \ln(k)$$

Et

$$n \times \ln (n)= \sum_{k=3n+1}^{4n} \ln(k) $$

En faite je bloque au niveau des indices, j'aimerais exprimer ces 3 sommes en 1 seule pour ensuite utiliser les sommes de Riemmman. La correction donne une autre méthode qui ne me plait pas beaucoup, je préfère de loin celle ci même si je bloque :p

Et Idem pour la seconde limite :

$$ \lim_{n\to +\infty} \left(\frac{(2n)!}{n^{n}\times n!}\right)^{\frac{1}{n}}$$

qui vaut :

$$=\exp(\dfrac{1}{n} \times (\ln(2n!)-\ln(n!)-n \times \ln(n) )$$

Or :

$$\ln(2n!)= \sum_{k=1}^{2n} \ln(k)$$
$$\ln(n!)= \sum_{k=1}^{n} \ln(k)$$

Et :

$$n \times \ln (n)= \sum_{k=n+1}^{2n} \ln(k) $$

idem , je bloque aussi au niveau des indices .

Veuillez m'excuser si les calculs ne s'affichent pas aussi bien que j'aimerais , je n'ai pas l'habitude décrire en LaTex .

Merci d'avance :)

Edit ; je comprends pas trop pourquoi tout s'affiche bien , si quelqu'un peut m'aider à les réécrire ca m'arrangerai . En gros la première c'est : Limite quand n tends vers +infini de (4n!)/(n^n * (3n!) ) à la puissance 1/n

Edit (Holosmos) : j'ai corrigé le mathjax

+0 -0

J'ai édité ton message pour corriger le Mathjax. J'ai pas bien compris le bug, mais c'est réglé.


Sur ta première limite, commence par faire attention au parentèsage, notamment $\ln(4n!)\neq \ln((4n)!)$.

Le problème que tu vas avoir, c'est effectivement que tes sommes vont pas être indicées de la même façon. Je comprends que cette méthode soit tentante, mais en fait elle est peu utilisable. Et là, comme ça, je vois mal comment arranger la chose pour continuer.

Merci beaucoup pour la correction Holosmos !

Je viens de remarquer mon erreur au niveau des parenthèses, sur ma feuille je les avaient pourtant bien notées :p

Par exemple sur ce pdf : http://gery.huvent.pagesperso-orange.fr/pdfbaggio/exosptsi/sommes_riemann.pdf

l'exercice 24.4 correspond à ma seconde limite à calculer et ils ont commencés à raisonner comme moi sauf que la suite je ne la comprends pas du tout :/

Sinon, comment toi tu aurais procéder pour calculer cette limite avec un niveau de maths L2 ? Car si tu as d'autres méthodes cela me va aussi c'est juste que intuitivement j'ai direct pensé à mon raisonnement plus haut mais ce n'est surement pas le meilleur.

Pour être plus précis, dans la correction que j'ai donné je ne comprend pas pourquoi on doit écrire cela :

$$n \times \ln (n)= \sum_{k=3n+1}^{4n} \ln(n) $$

Et pas cela :

$$n \times \ln (n)= \sum_{k=3n+1}^{4n} \ln(k) $$

La première somme ne veut rien dire , on fait une somme ou k varie de 3n+1 à 4n sauf que dans la somme on a pas de k :/

Si quelqu'un pouvait m'expliquer pourquoi on peut et on doit écrire la première somme plutôt que la seconde ca m'aiderait beaucoup ! Car après en trouvant :

$$n \times \ln (n)= \sum_{k=3n+1}^{4n} \ln(n) $$

la suite découle tout seule sans problème.

+0 -0

Parce que le premier est juste, et l'autre faux. :P Ta deuxième équation est tout simplement fausse. Pas contre, la première,

$$ n \times \ln(n) = \ln(n)+\ln(n)+...+\ln(n)$$

La somme étant faite n fois.

$$ = \sum_{i=1}^n \ln(n) $$

puis par décalage des indice,

$$= \sum_{i=3n+1}^{4n} \ln(n)$$

L'intérêt de faire comme ça, c'est que comme tu as déjà exprimé $\ln((4n)!)-\ln((3n)!)$ comme une somme de $3n+1$ à $4n$, tu peux t'en sortir.

+1 -0

Parce que le premier est juste, et l'autre faux. :P Ta deuxième équation est tout simplement fausse. Pas contre, la première,

$$ n \times \ln(n) = \ln(n)+\ln(n)+...+\ln(n)$$

La somme étant faite n fois.

$$ = \sum_{i=1}^n \ln(n) $$

puis par décalage des indice,

$$= \sum_{i=3n+1}^{4n} \ln(n)$$

L'intérêt de faire comme ça, c'est que comme tu as déjà exprimé $\ln((4n)!)-\ln((3n)!)$ comme une somme de $3n+1$ à $4n$, tu peux t'en sortir.

Gabbro

En faite je comprend que n*ln (n) c'est ln (n)+ln(n)+…+ln(n)

Juste en terme d'indice et de somme ca me chiffone un petit peu :/ comment on peut dire qu'on fait varier k de 3n+1 à 4n alors qu'il n'existe aucun k dans notre somme ?

Connectez-vous pour pouvoir poster un message.
Connexion

Pas encore membre ?

Créez un compte en une minute pour profiter pleinement de toutes les fonctionnalités de Zeste de Savoir. Ici, tout est gratuit et sans publicité.
Créer un compte